Is there a proof of an infinite number of prime numbers using the irrationality of $e$?











up vote
4
down vote

favorite












That the set of prime integers is infinite can be proved using the irrationality of $pi$; see this wikipedia link. It analyzes the representation



$tag 1 {displaystyle {frac {pi }{4}}={frac {3}{4}}times {frac {5}{4}}times {frac {7}{8}}times {frac {11}{12}}times {frac {13}{12}}times {frac {17}{16}}times {frac {19}{20}}times {frac {23}{24}}times {frac {29}{28}}times {frac {31}{32}}times cdots }$



It 'seems only fair' that the same thing can be done using $e$:




Question 1: Has a proof that there are an infinite number of primes been
constructed that uses the irrationality of $e$?




If the existence of such a proof is not available, then




Question 1-1: Is a proof available using any properties of $e$?




If again, no answers, then




Question 2: Can someone show that the set of prime integers is
infinite using properties of Euler's constant?




My work



I found this representation of $e$,



$tag 2 {displaystyle e=1+{cfrac {2}{1+{cfrac {1}{6+{cfrac {1}{10+{cfrac {1}{14+{cfrac {1}{18+{cfrac {1}{22+{cfrac {1}{26+ddots ,}}}}}}}}}}}}}}.}$



Note that with $text{(1)}$ there is a 'corresponding' sequence



$tag 3 4,8,12,16,dots$



and that with $text{(2)}$ we can see



$tag 4 6, 10,14,18,dots$



I just find this interesting; perhaps it supplies a 'connection'.



We know that eventually every prime number will divide a term in the $text{(4)}$ sequence.



If the only primes are contained in the finite set $mathcal P = {2,3,5, dots, p}$, then let $beta = 2 times 3 times 5 times dots times p$.



If we analyze $text{(2)}$ using base $beta$, the expansion of $e$ would eventually be repetitive or simply terminate.



(the statement in bold is an unproven bold statement)










share|cite|improve this question






















  • It's hard to see why one would want to prove it that way, as the usual $1+prod_{i=1}^n p_i$ argument does it easily.
    – Richard Martin
    Nov 22 at 14:46












  • Or, better still, try and figure it out from what I have just said.
    – Richard Martin
    Nov 22 at 14:50










  • @RichardMartin Ok - Euclid's proof. Just interested in seeing a 'later stage' proof.
    – CopyPasteIt
    Nov 22 at 14:52










  • @RichardMartin Someone putting up the wiki article thought it was interesting enough to put down the $pi$ proof...
    – CopyPasteIt
    Nov 22 at 14:53






  • 1




    @RichardMartin More proofs = more fun. But seriously, new proofs to existing theorems are interesting because of any novel ideas they might contain, not because they achieve the same conclusion.
    – Klangen
    Nov 23 at 9:12















up vote
4
down vote

favorite












That the set of prime integers is infinite can be proved using the irrationality of $pi$; see this wikipedia link. It analyzes the representation



$tag 1 {displaystyle {frac {pi }{4}}={frac {3}{4}}times {frac {5}{4}}times {frac {7}{8}}times {frac {11}{12}}times {frac {13}{12}}times {frac {17}{16}}times {frac {19}{20}}times {frac {23}{24}}times {frac {29}{28}}times {frac {31}{32}}times cdots }$



It 'seems only fair' that the same thing can be done using $e$:




Question 1: Has a proof that there are an infinite number of primes been
constructed that uses the irrationality of $e$?




If the existence of such a proof is not available, then




Question 1-1: Is a proof available using any properties of $e$?




If again, no answers, then




Question 2: Can someone show that the set of prime integers is
infinite using properties of Euler's constant?




My work



I found this representation of $e$,



$tag 2 {displaystyle e=1+{cfrac {2}{1+{cfrac {1}{6+{cfrac {1}{10+{cfrac {1}{14+{cfrac {1}{18+{cfrac {1}{22+{cfrac {1}{26+ddots ,}}}}}}}}}}}}}}.}$



Note that with $text{(1)}$ there is a 'corresponding' sequence



$tag 3 4,8,12,16,dots$



and that with $text{(2)}$ we can see



$tag 4 6, 10,14,18,dots$



I just find this interesting; perhaps it supplies a 'connection'.



We know that eventually every prime number will divide a term in the $text{(4)}$ sequence.



If the only primes are contained in the finite set $mathcal P = {2,3,5, dots, p}$, then let $beta = 2 times 3 times 5 times dots times p$.



If we analyze $text{(2)}$ using base $beta$, the expansion of $e$ would eventually be repetitive or simply terminate.



(the statement in bold is an unproven bold statement)










share|cite|improve this question






















  • It's hard to see why one would want to prove it that way, as the usual $1+prod_{i=1}^n p_i$ argument does it easily.
    – Richard Martin
    Nov 22 at 14:46












  • Or, better still, try and figure it out from what I have just said.
    – Richard Martin
    Nov 22 at 14:50










  • @RichardMartin Ok - Euclid's proof. Just interested in seeing a 'later stage' proof.
    – CopyPasteIt
    Nov 22 at 14:52










  • @RichardMartin Someone putting up the wiki article thought it was interesting enough to put down the $pi$ proof...
    – CopyPasteIt
    Nov 22 at 14:53






  • 1




    @RichardMartin More proofs = more fun. But seriously, new proofs to existing theorems are interesting because of any novel ideas they might contain, not because they achieve the same conclusion.
    – Klangen
    Nov 23 at 9:12













up vote
4
down vote

favorite









up vote
4
down vote

favorite











That the set of prime integers is infinite can be proved using the irrationality of $pi$; see this wikipedia link. It analyzes the representation



$tag 1 {displaystyle {frac {pi }{4}}={frac {3}{4}}times {frac {5}{4}}times {frac {7}{8}}times {frac {11}{12}}times {frac {13}{12}}times {frac {17}{16}}times {frac {19}{20}}times {frac {23}{24}}times {frac {29}{28}}times {frac {31}{32}}times cdots }$



It 'seems only fair' that the same thing can be done using $e$:




Question 1: Has a proof that there are an infinite number of primes been
constructed that uses the irrationality of $e$?




If the existence of such a proof is not available, then




Question 1-1: Is a proof available using any properties of $e$?




If again, no answers, then




Question 2: Can someone show that the set of prime integers is
infinite using properties of Euler's constant?




My work



I found this representation of $e$,



$tag 2 {displaystyle e=1+{cfrac {2}{1+{cfrac {1}{6+{cfrac {1}{10+{cfrac {1}{14+{cfrac {1}{18+{cfrac {1}{22+{cfrac {1}{26+ddots ,}}}}}}}}}}}}}}.}$



Note that with $text{(1)}$ there is a 'corresponding' sequence



$tag 3 4,8,12,16,dots$



and that with $text{(2)}$ we can see



$tag 4 6, 10,14,18,dots$



I just find this interesting; perhaps it supplies a 'connection'.



We know that eventually every prime number will divide a term in the $text{(4)}$ sequence.



If the only primes are contained in the finite set $mathcal P = {2,3,5, dots, p}$, then let $beta = 2 times 3 times 5 times dots times p$.



If we analyze $text{(2)}$ using base $beta$, the expansion of $e$ would eventually be repetitive or simply terminate.



(the statement in bold is an unproven bold statement)










share|cite|improve this question













That the set of prime integers is infinite can be proved using the irrationality of $pi$; see this wikipedia link. It analyzes the representation



$tag 1 {displaystyle {frac {pi }{4}}={frac {3}{4}}times {frac {5}{4}}times {frac {7}{8}}times {frac {11}{12}}times {frac {13}{12}}times {frac {17}{16}}times {frac {19}{20}}times {frac {23}{24}}times {frac {29}{28}}times {frac {31}{32}}times cdots }$



It 'seems only fair' that the same thing can be done using $e$:




Question 1: Has a proof that there are an infinite number of primes been
constructed that uses the irrationality of $e$?




If the existence of such a proof is not available, then




Question 1-1: Is a proof available using any properties of $e$?




If again, no answers, then




Question 2: Can someone show that the set of prime integers is
infinite using properties of Euler's constant?




My work



I found this representation of $e$,



$tag 2 {displaystyle e=1+{cfrac {2}{1+{cfrac {1}{6+{cfrac {1}{10+{cfrac {1}{14+{cfrac {1}{18+{cfrac {1}{22+{cfrac {1}{26+ddots ,}}}}}}}}}}}}}}.}$



Note that with $text{(1)}$ there is a 'corresponding' sequence



$tag 3 4,8,12,16,dots$



and that with $text{(2)}$ we can see



$tag 4 6, 10,14,18,dots$



I just find this interesting; perhaps it supplies a 'connection'.



We know that eventually every prime number will divide a term in the $text{(4)}$ sequence.



If the only primes are contained in the finite set $mathcal P = {2,3,5, dots, p}$, then let $beta = 2 times 3 times 5 times dots times p$.



If we analyze $text{(2)}$ using base $beta$, the expansion of $e$ would eventually be repetitive or simply terminate.



(the statement in bold is an unproven bold statement)







soft-question prime-numbers alternative-proof continued-fractions eulers-constant






share|cite|improve this question













share|cite|improve this question











share|cite|improve this question




share|cite|improve this question










asked Nov 22 at 14:40









CopyPasteIt

3,9421627




3,9421627












  • It's hard to see why one would want to prove it that way, as the usual $1+prod_{i=1}^n p_i$ argument does it easily.
    – Richard Martin
    Nov 22 at 14:46












  • Or, better still, try and figure it out from what I have just said.
    – Richard Martin
    Nov 22 at 14:50










  • @RichardMartin Ok - Euclid's proof. Just interested in seeing a 'later stage' proof.
    – CopyPasteIt
    Nov 22 at 14:52










  • @RichardMartin Someone putting up the wiki article thought it was interesting enough to put down the $pi$ proof...
    – CopyPasteIt
    Nov 22 at 14:53






  • 1




    @RichardMartin More proofs = more fun. But seriously, new proofs to existing theorems are interesting because of any novel ideas they might contain, not because they achieve the same conclusion.
    – Klangen
    Nov 23 at 9:12


















  • It's hard to see why one would want to prove it that way, as the usual $1+prod_{i=1}^n p_i$ argument does it easily.
    – Richard Martin
    Nov 22 at 14:46












  • Or, better still, try and figure it out from what I have just said.
    – Richard Martin
    Nov 22 at 14:50










  • @RichardMartin Ok - Euclid's proof. Just interested in seeing a 'later stage' proof.
    – CopyPasteIt
    Nov 22 at 14:52










  • @RichardMartin Someone putting up the wiki article thought it was interesting enough to put down the $pi$ proof...
    – CopyPasteIt
    Nov 22 at 14:53






  • 1




    @RichardMartin More proofs = more fun. But seriously, new proofs to existing theorems are interesting because of any novel ideas they might contain, not because they achieve the same conclusion.
    – Klangen
    Nov 23 at 9:12
















It's hard to see why one would want to prove it that way, as the usual $1+prod_{i=1}^n p_i$ argument does it easily.
– Richard Martin
Nov 22 at 14:46






It's hard to see why one would want to prove it that way, as the usual $1+prod_{i=1}^n p_i$ argument does it easily.
– Richard Martin
Nov 22 at 14:46














Or, better still, try and figure it out from what I have just said.
– Richard Martin
Nov 22 at 14:50




Or, better still, try and figure it out from what I have just said.
– Richard Martin
Nov 22 at 14:50












@RichardMartin Ok - Euclid's proof. Just interested in seeing a 'later stage' proof.
– CopyPasteIt
Nov 22 at 14:52




@RichardMartin Ok - Euclid's proof. Just interested in seeing a 'later stage' proof.
– CopyPasteIt
Nov 22 at 14:52












@RichardMartin Someone putting up the wiki article thought it was interesting enough to put down the $pi$ proof...
– CopyPasteIt
Nov 22 at 14:53




@RichardMartin Someone putting up the wiki article thought it was interesting enough to put down the $pi$ proof...
– CopyPasteIt
Nov 22 at 14:53




1




1




@RichardMartin More proofs = more fun. But seriously, new proofs to existing theorems are interesting because of any novel ideas they might contain, not because they achieve the same conclusion.
– Klangen
Nov 23 at 9:12




@RichardMartin More proofs = more fun. But seriously, new proofs to existing theorems are interesting because of any novel ideas they might contain, not because they achieve the same conclusion.
– Klangen
Nov 23 at 9:12















active

oldest

votes











Your Answer





StackExchange.ifUsing("editor", function () {
return StackExchange.using("mathjaxEditing", function () {
StackExchange.MarkdownEditor.creationCallbacks.add(function (editor, postfix) {
StackExchange.mathjaxEditing.prepareWmdForMathJax(editor, postfix, [["$", "$"], ["\\(","\\)"]]);
});
});
}, "mathjax-editing");

StackExchange.ready(function() {
var channelOptions = {
tags: "".split(" "),
id: "69"
};
initTagRenderer("".split(" "), "".split(" "), channelOptions);

StackExchange.using("externalEditor", function() {
// Have to fire editor after snippets, if snippets enabled
if (StackExchange.settings.snippets.snippetsEnabled) {
StackExchange.using("snippets", function() {
createEditor();
});
}
else {
createEditor();
}
});

function createEditor() {
StackExchange.prepareEditor({
heartbeatType: 'answer',
convertImagesToLinks: true,
noModals: true,
showLowRepImageUploadWarning: true,
reputationToPostImages: 10,
bindNavPrevention: true,
postfix: "",
imageUploader: {
brandingHtml: "Powered by u003ca class="icon-imgur-white" href="https://imgur.com/"u003eu003c/au003e",
contentPolicyHtml: "User contributions licensed under u003ca href="https://creativecommons.org/licenses/by-sa/3.0/"u003ecc by-sa 3.0 with attribution requiredu003c/au003e u003ca href="https://stackoverflow.com/legal/content-policy"u003e(content policy)u003c/au003e",
allowUrls: true
},
noCode: true, onDemand: true,
discardSelector: ".discard-answer"
,immediatelyShowMarkdownHelp:true
});


}
});














draft saved

draft discarded


















StackExchange.ready(
function () {
StackExchange.openid.initPostLogin('.new-post-login', 'https%3a%2f%2fmath.stackexchange.com%2fquestions%2f3009218%2fis-there-a-proof-of-an-infinite-number-of-prime-numbers-using-the-irrationality%23new-answer', 'question_page');
}
);

Post as a guest















Required, but never shown






























active

oldest

votes













active

oldest

votes









active

oldest

votes






active

oldest

votes
















draft saved

draft discarded




















































Thanks for contributing an answer to Mathematics Stack Exchange!


  • Please be sure to answer the question. Provide details and share your research!

But avoid



  • Asking for help, clarification, or responding to other answers.

  • Making statements based on opinion; back them up with references or personal experience.


Use MathJax to format equations. MathJax reference.


To learn more, see our tips on writing great answers.





Some of your past answers have not been well-received, and you're in danger of being blocked from answering.


Please pay close attention to the following guidance:


  • Please be sure to answer the question. Provide details and share your research!

But avoid



  • Asking for help, clarification, or responding to other answers.

  • Making statements based on opinion; back them up with references or personal experience.


To learn more, see our tips on writing great answers.




draft saved


draft discarded














StackExchange.ready(
function () {
StackExchange.openid.initPostLogin('.new-post-login', 'https%3a%2f%2fmath.stackexchange.com%2fquestions%2f3009218%2fis-there-a-proof-of-an-infinite-number-of-prime-numbers-using-the-irrationality%23new-answer', 'question_page');
}
);

Post as a guest















Required, but never shown





















































Required, but never shown














Required, but never shown












Required, but never shown







Required, but never shown

































Required, but never shown














Required, but never shown












Required, but never shown







Required, but never shown







Popular posts from this blog

Quarter-circle Tiles

build a pushdown automaton that recognizes the reverse language of a given pushdown automaton?

Mont Emei